您當前的位置:首頁 > 文化

利用柯西不等式來配湊係數

作者:由 Sensitives 發表于 文化時間:2021-07-14

利用恆等式求代數式的值是多少

ps:本文僅適合聯賽難度(確信)

回顧一下

柯西不等式:

設a_{i},b_{i}\in R,i=1,\cdots,n,則

 (\sum_{i=1}^{n}{a_{i}^{2}})(\sum_{i=1}^{n}{b_{i}^{2}})\geq(\sum_{i=1}^{n}{a_{i}b_{i}})^{2}\large\

當且僅當a_{i}=kb_{i}(k\ne0),i=1,...,n時取等

Ⅰ 起步

柯西不等式的取等條件是很多競賽生(特別是初學不等式者)所不曾注意的,事實上利用柯西不等式待定係數的核心就是在於它的取等條件,下面我們來看幾個例子:

例一:

(IMO Short List 2016)求最大的實數

a,

使得對所有正整數

n\geq1

和任意

x_{0},x_{1},\cdots x_{n}

滿足

0=x_{0}<x_{1}<x_{2}<\cdots <x_{n},

均有

\sum_{k=1}^{n}{\frac{1}{x_{k}-x_{k-1}}}\geq a\sum_{k=1}^{n}{\frac{k+1}{x_{i}}} \cdots(1)

分析與解答:

由原不等式直接估計

a

的上界是一件困難的事,為了構造出

\frac{1}{x_{i}-x_{i-1}}

這一項,我們來考慮待定係數,由柯西不等式,有:(其中

a_{k}

為待定係數)

\left[x_{k-1}+(x_{k}-x_{k-1}) \right]\left(\frac{a_{k-1}^{2}}{x_{k-1}}+\frac{1}{x_{k}-x_{k-1}}\right)\geq (a_{k-1}+1)^{2}

於是

\frac{1}{x_{k}-x_{k-1}}\geq \frac{(a_{k-1}+1)^{2}}{x_{k}}-\frac{a_{k-1}^{2}}{x_{k-1}}

將其餘(n-1)個式子相加,比較

\frac{1}{x_{k}}

的係數,有

\left((a_{k-1}+1)^2-a_{k}^2)=\right)(a_{k-1}+1-a_{k})(a_{k-1}+1+a_{k})=a(k+1)

其中

a_{0}=0

(因為

x_{0}=0

)。如果把

k

看做變數,那麼上式右邊為關於

k

的一次式,因此我們大膽猜測

\left\{ a_{k}\right\}

可能是

等差數列

a_{k}=bk

(b為常數),有:

(1-b)(2bk-b+1)=a(k+1)

為湊出

a(k+1)

這一項,上式左邊

k

和常數項係數必須相等,因此

2b=1-b\Rightarrow b=\frac{1}{3}

回到原題,則當

n\geq k\geq2

時,有:

\left[x_{k-1}+(x_{k}-x_{k-1}) \right]\left(\frac{(k-1)^{2}}{x_{k-1}}+\frac{3^2}{x_{k}-x_{k-1}}\right)\geq (k-1+3)^{2}\cdots(2)

\Rightarrow \frac{3^2}{x_{k}-x_{k-1}}\geq \frac{(k+2)^{2}}{x_{k}}-\frac{(k-1)^2}{x_{k-1}}

\Rightarrow        \frac{3^2}{x_{1}-x_{0}}+\sum_{k=2}^{n}\frac{3^2}{x_{k}-x_{k-1}} \geq\ \frac{3^2}{x_{1}}+\sum_{k=2}^{n}\left[\frac{(k+2)^{2}}{x_{k}}-\frac{(k-1)^2}{x_{k-1}}\right]

\Rightarrow9 \cdot\sum_{k=1}^{n}\frac{1}{x_{k}-x_{k-1}} \geq\ 4\cdot \sum_{k=1}^{n}\frac{k+1}{x_{k}}+\frac{n^2}{x_{n}}

,故

a=\frac{4}{9}

時,不等式恆成立

下證

a_{max}=\frac{4}{9}

,由(2)的取等條件得

3x_{k-1}=(k-1)(x_{k}-x_{k-1})

(k-1)x_{k}=(k-2)x_{k-1}\Rightarrow \frac{x_{k}}{(k+2)(k+1)k}=\frac{x_{k-1}}{(k-1)(k+1)k}

為了讓(差分後)形式好看點,我們不妨取

x_{k}=\frac{1}{3}k(k+1)(k+2)

,則

(1)式的左邊

LHS=\sum_{k=1}^{n}\frac{1}{k(k+1)}=1-\frac{1}{n+1}

(1)式的右邊

RHS=a\sum_{k=1}^{n}{\frac{k+1}{x_{i}}}=3a\sum_{k=1}^{n}{\frac{1}{k(k+2)}}=\frac{3}{2}a\left(1+\frac{1}{2}-\frac{1}{n+1}-\frac{1}{n+2}\right)

n\rightarrow \infty,2\geq3a\times\frac{3}{2}\Rightarrow a\leq\frac{4}{9}

,從而

a_{max}=\frac{4}{9}\

\square

例二:

(《不等式的秘密》)對任意實數

x_{1},x_{2},\cdots,x_{n},

x_{1}^2+(x_{1}+x_{2})^2+\cdots+(x_{1}+x_{2}+\cdots+x_{n})^2\leq t(x_{1}^2+x_{2}^2+\cdots+x_{n}^2)\cdots(3)

恆成立,求

t

的最小值

分析:

不等式左邊每一個

x_{i}

地位都不相等,因此我們考慮待定係數

c_{i}>0(i=1,2,\cdots,n)

,由柯西不等式,得

(x_{1}+x_{2}+\cdots+x_{k})^2\leq (c_{1}+c_{2}+\cdots+c_{k})(\frac{x_{1}^2}{c_{1}}+\frac{x_{2}^2}{c_{2}}+\cdots+\frac{x_{k}^2}{c_{k}})\\

上式對

k=1,2,\cdots,n

求和,得

x_{1}^2+(x_{1}+x_{2})^2+\cdots+(x_{1}+x_{2}+\cdots+x_{n})^2\leq \frac{(s_{1}+s_{2}+\cdots+s_{n})x_{1}^2}{c_{1}}+\frac{(s_{2}+\cdots+s_{n})x_{2}^2}{c_{2}}+\cdots+\frac{s_{n}x_{n}^2}{c_{n}}\cdots(4)

其中

s_{k}=\sum_{i=1}^{k}{c_{i}}

,為了構造出(3)式的形式,我們希望(4)式的右邊每個

x_{k}

的係數相等,利用這一點,我們來選取合適的

c_{k}

,即

\frac{(s_{1}+s_{2}+\cdots+s_{n})}{c_{1}}=\frac{(s_{2}+\cdots+s_{n})}{c_{2}}=\cdots=\frac{s_{n}}{c_{n}}\\

等價於

(

其中

c_{n+1}=0)

\frac{s_{1}}{c_{1}-c_{2}}=\frac{s_{2}}{c_{2}-c_{3}}=\cdots=\frac{s_{n-1}}{c_{n-1}-c_{n}}=\frac{s_{n}}{c_{n}-c_{n+1}}\\

c_{k}=s_{k}-s_{k-1}

,上式又等價於 (其中

s_{n+1}=\sum_{i=1}^{n+1}{c_{i}}

\frac{s_{2}}{s_{1}}=\frac{s_{1}+s_{2}}{s_{2}}=\cdots=\frac{s_{n-2}+s_{n}}{s_{n-1}}=\frac{s_{n-1}+s_{n+1}}{s_{n}}\\

\frac{s_{k+1}}{s_{k}}=\lambda_{k}

,則

\lambda_{k+1}=\lambda_{1}-\frac{1}{\lambda_{k}}(1\leq k \leq n-1)

,如果我們熟悉三角公式,不難想到:取

\lambda_{1}=2\cos{\frac{π}{2n+1}}=\frac{sin\frac{2\pi}{2n+1}}{sin\frac{\pi}{2n+1}}

,則由歸納法和上述的約束條件易得

\lambda_{k}=\frac{sin\frac{(k+1)\pi}{2n+1}}{sin\frac{k\pi}{2n+1}}

,故我們取

s_{k}=\sin{k\alpha}

,其中

\alpha=\frac{\pi}{2n+1}

,那麼我們有

c_{k}=\sin{k\alpha}-\sin{(k-1)\alpha}

,則

\begin{align}s_{k}+s_{k+1}+\cdots+s_{n}&=\sum_{i=k}^{n}{\sin{i\alpha}}\\&=\frac{\cos{\left(k-\frac{1}{2}\right)\alpha}-cos(n+\frac{1}{2})\alpha}{2\sin{\frac{\alpha}{2}}}\\&=\frac{\cos{\left(k-\frac{1}{2}\right)\alpha}}{2\sin{\frac{\alpha}{2}}}\end{align}\\

於是

t=\frac{s_{k}+s_{k+1}+\cdots+s_{n}}{c_{k}}=\frac{\cos{\left(k-\frac{1}{2}\right)\alpha}}{2\sin{\frac{\alpha}{2}}(\sin{k\alpha}-\sin{(k-1)\alpha})}=\frac{1}{4\sin^2{\frac{\alpha}{2}}}\\

由(4)的取等條件知

\frac{x_{1}}{\sin{\alpha}}=\frac{x_{2}}{\sin{2\alpha}-\sin{\alpha}}=\cdots=\frac{x_{n}}{c_{k}=\sin{n\alpha}-\sin{(n-1)\alpha}}\\

代入即得

t=\frac{1}{4\sin^2{\frac{\pi}{2(2n+1)}}}

\square

注:

類似的,還有反向不等式

x_{1}^2+(x_{1}+x_{2})^2+\cdots+(x_{1}+x_{2}+\cdots+x_{n})^2\geq \frac{1}{4\cos^2{\frac{\pi}{2n+1}}}(x_{1}^2+x_{2}^2+\cdots+x_{n}^2)\\

證明留給讀者

Ⅱ 應用

透過上面幾個例子,相信大家都對這種方法有了深刻的感受,下面我們再來看幾個例子:

例三:

(2018巴爾幹MO預選題)對任意正實數

x_{1},x_{2},\cdots x_{n}

,證明:

\sum_{i=1}^{n}x_i^2\geq\frac{1}{n+1}(\sum_{i=1}^{n}x_i)^2+\frac{12(\sum_{i=1}^{n}ix_i)^2}{n(n+1)(n+2)(3n+1)}

分析和證明:

不等式的右邊既不輪換也不對稱,為了配湊出

\frac{12(\sum_{i=1}^{n}ix_i)^2}{n(n+1)(n+2)(3n+1)}

,我們來考慮

\sum_{i=1}^{n}x_i^2-\frac{1}{n+1}(\sum_{i=1}^{n}x_i)^2\\=\sum_{i=1}^{n}\left({x_{i}-\frac{1}{n+1}\sum_{i=1}^{n}x_i}\right)^2+\left(\frac{1}{n+1}\sum_{i=1}^{n}x_i\right)^2

因此原不等式

\Leftrightarrow\sum_{i=1}^{n}\left({x_{i}-\frac{1}{n+1}\sum_{i=1}^{n}x_i}\right)^2+\left(\frac{1}{n+1}\sum_{i=1}^{n}x_i\right)^2\geq\frac{12(\sum_{i=1}^{n}ix_i)^2}{n(n+1)(n+2)(3n+1)}\cdots(5)

S=\frac{1}{n+1}\sum_{i=1}^{n}x_i,a_{i}=x_{i}-S

,則(5)式的左右兩邊:

LHS^{(5)}=\sum_{i=1}^{n}{a_{i}}^2+S^2

RHS^{(5)}=\frac{12(\sum_{i=1}^{n}ia_i+\frac{n(n+1)}{2}S)^2}{n(n+1)(n+2)(3n+1)}

根據取等,不難由柯西不等式得出係數:

(\sum_{i=1}^{n}{a_{i}}^2+S^2)(1^2+2^2+\cdots+n^2+\frac{n^2(n+1)^2}{2^2})\geq(\sum_{i=1}^{n}ia_i+\frac{n(n+1)}{2}S)^2\\

又由

1^2+2^2+\cdots+n^2+\frac{n^2(n+1)^2}{4}\\=\frac{n(n+1)(2n+1)}{6}+\frac{n^2(n+1)^2}{4}\\=\frac{12}{n(n+1)(n+2)(3n+1)}\qquad\square

注:

事實上:

\frac{12}{n(n+1)(n+2)(3n+1)}=\frac{1}{\sum_{i=1}^{n}{i^2}+\sum_{i=1}^{n}{i^3}}

例四:

(2017臺灣TST)給定正整數

n\geq3

。求最大的實數

\lambda

,使得只要正數

a_{1},a_{2},\cdots,a_{n}

滿足

\sum_{i=1}^{n}a_i^2<\lambda(\sum_{i=1}^{n}a_i)^2

,就有

a_1,a_2,\cdots,a_n

中任意三個數均可作為某個三角形的邊長

分析與解答:

我們來考慮這個問題的反面(逆否命題):求最大的實數

\lambda

,使得若存在

a_i,a_j,a_k

,且

a_i\geq a_j+a_k

,則必有

\sum_{i=1}^{n}a_i^2\geq \lambda(\sum_{i=1}^{n}a_i)^2

成立 於是我們選擇取

a_1=2,a_2=a_3=1,a_4=\cdots=a_n=x

,則令

f(x)=\frac{\sum_{i=1}^{n}a_i^2}{(\sum_{i=1}^{n}a_i)^2}=\frac{2^2+1^2+1^2+x^2(n-3)}{[2+1+1+x(n-3)]^2}\\

\Rightarrow f

,再令

f

回到原題,一方面我們取

a_1=4,a_2=a_3=2,a_4=\cdots=a_n=3,

\lambda\leq \frac{16+4+4+x^2(n-3)}{(8+3(n-3))^2}=\frac{3}{3n-1}\\

下證:若

a_1\geq a_2+a_3

,則

\sum_{i=1}^{n}a_i^2\geq \frac{3}{3n-1}(\sum_{i=1}^{n}a_i)^2

簡證:因為我們已經知道了不等式的取等條件,所以由柯西不等式:

\begin{align}\sum_{i=1}^na_i^2&\geq \frac{(4a_1+2a_2+2a_3+3a_4+...+3a_n)^2}{4^2+2^2+2^2+3^2+...+3^2}\\ &\geq\frac{(3a_1+3a_2+3a_3+3a_4+...+3a_n)^2}{4^2+2^2+2^2+3^2+...+3^2}\\&=\frac{3}{3n-1}(\sum_{i=1}^na_i)^2\end{align}\\

\lambda

最大值為

\frac{3}{3n-1}

\square

例五:

設正整數

n\geq2

,實數

x_i(i=1,2,\cdots,n)

滿足

x_1\geq x_2\geq \cdots\geq x_n,

\left\{ \begin{array}   xx_1+x_2+\cdots+x_n=0 &\\ x_1^2+x_2^2+\cdots+x_n^2=n(n-1)\end{array} \right.

x_1+x_2

的最大值和最小值

分析與解答:

一次式求最大值是比較容易的,所以我們先求

x_1+x_2

的最大值:

由柯西:

\begin{align}n(n-1)&=x_1^2+x_2^2+(x_3^2+···+x_n^2)\\&\geq\frac{(x_1+x_2)^2}{2}+\frac{(x_3+···+x_n)^2}{n-2}\\&=\frac{n(x_1+x_2)^2}{2(n-2)}\end{align}

\Rightarrow \quad (x_1+x_2)^2\leq2(n-1)(n-2)\qquad \Rightarrow x_1+x_2\leq\sqrt{2(n-1)(n-2)}

當且僅當

\left\{ \begin{array}{rl} x_1=x_2=\sqrt{\frac{(n-1)(n-2)}{2}}\\ x_3=x_4=···=x_n=-\sqrt{\frac{2(n-1)}{n}} \end{array} \right.

時取等

下面我們來求

x_1+x_2

的最小值。,一般來說一次式的最小值常用均值不等式求解,但這裡並沒有任何乘積的形式(並且

x_i

不全是正數),如果我們先考慮一個簡單的情形:

在條件

\left\{ \begin{array}{rl} a+b+c=0\cdots(a)\\a^2+b^2+c^2=6\cdots(b)\\a\geq b\geq c\cdots(c) \end{array} \right.

下,求

a+b

的最小值

然後該怎麼做呢?我們目前並不能充分使用到(a)和(c),所以我們先來考慮(b),去嘗試讓

(a+b)^2

大於等於一個二次式,我們希望這個二次式最好能直接放縮成

a^2+b^2+c^2

的若干倍,但

a,b

c

的地位天生不等,所以不可能只包含

a^2+b^2+c^2

這一項。由於

a+b

很難直接放縮成有關

c

的式子,為了防止放過,我們來嘗試配湊恆等式。

我們考慮

(a+b)^2=a^2+b^2+c^2

再加上一個式子,這個式子它得是二次的,還必須是具有

a,b

地位相等且與

c

地位不相等的性質。這讓我們聯想到

(a+c)(b+c)

(a-c)(b-c)

這兩個式子,但前者無法進一步放縮(因為無法判斷正負),因此我們考慮

(a-c)(b-c)=ab-bc-ca+c^2

,為了再把

(a+b)^2

中的

ab

消掉,我們不妨再給

(a-c)(b-c)

配一個係數2。

此時右邊有關

c

的代數式為

-2ac-2bc+3c^2

,為了把有關

c

的項消掉,我們考慮再去加一項,而這一項必須非負,且有關

c^2

的項係數為負,由這兩點我們就可以想到去配一個有關

[x(a+b)-yc]

的項。但不能是

[x(a+b)-yc]^2

,因為這一項是不能放成0的,因為如果放成

0

,結合之前新增的

(a-c)(b-c)

,我們得出

a=b=c

,這顯然是錯誤的。不過值得一提的是,我們能透過調整x,y使這一項恆正,這就啟示我們去再用一個可以放成

0

的一次式去與其相乘,於是我們就想到了一直沒使用過的(a),設:

\lambda(a+b)^2=a^2+b^2+c^2+2(a-c)(b-c)+(a+b+c)[x(a+b)-yc]

為了消掉

3c^2

,令

y=3

,展開可得

\lambda(a+b)^2=(x+1)(a+b)^2+(x-5)(ac+bc)

,於是

x=5

從而

6(a+b)^2=a^2+b^2+c^2+2(a-c)(b-c)+(a+b+c)[5(a+b)-3c]\geq 6

於是,當

n=3

時,

a+b\geq 1

,當

a=2,b=c=-1

時取等

有了三元的基礎,當

n\geq 4

時,我們同理可得(大霧):

n(n-1)(x_1+x_2)^2=4\sum_{i=1}^{n}{x_i^2}+4\sum_{i\ne j}(x_1-x_i)(x_2-x_j) \\+4\sum_{i=1}^{n}{x_i}\left[(n-2)(x_1+x_2)-\sum_{i=3}^{n}{x_i}\right]+(n-1)(n-4)(x_1-x_2)^2\geq 4n(n-1)\\

於是,當

n\geq 4

時,

x_1+x_2\geq 2

,取等條件:當

x_k=1(1\leq k\leq n-1)

x_n=1-n

時取等

Ⅲ 習題

這些題目是我從各種地方找來的題,題目數量可能有點多,而且有的題目也並不是只用到了柯西不等式,不過我並沒有放出解答,目的是希望大家能認真思考(qi shi shi lan)

1。(《不等式的秘密》)設

x,y,z>0

,且滿足

x+y+z=3

,求

x^4+2y^4+3z^4

的最小值

2。(羊明亮)給定實數

p_1,p_2,\cdots,p_n

,滿足

\sum_{i=1}^{n}{p_{i}x_{i}}=1

的實數

x_1,x_2,\cdots,x_n

,求

\sum_{i=1}^{n}{x_{i}^{2}}+\left(\sum_{i=1}^{n}{x_{i}}\right)^{2}\\

的最小值

3。給定整數

n\geq2

,求最小的實數

\lambda

,使得對任意滿足

\sum_{i=1}^nia_i=0

的實數

a_1,a_2,···,a_n

,都有

(\sum_{i=1}^na_i)^2\leq\lambda\sum_{i=1}^na_i^2

4。設

x_i>0,i=1,2,\cdots,n,x_{n+1}=x_1

,證明 :

\sum_{i=1}^n\frac{x_i^2}{x_{i+1}}\geq\sum_{i=1}^nx_i+\frac{4(x_1-x_n)^2}{\sum_{i=1}^nx_i}

5。(RMO 2007)對於

a_i\in R^+,1\leq i\leq n

,證明 :

(1+a_1)(1+a_1+a_2)···(1+a_1+a_2+···+a_n)\geq\sqrt{(n+1)^{n+1}a_1a_2···a_n}

6。設整數

n\geq 2

,實數

a_1,a_2,\cdots,a_n

滿足

\sum_{i=1}^{n}{a_i}=1

,設

b_k=\left(1-\frac{1}{4^k}\right)·\sqrt{a_1^2+a_2^2+···+a_k^2}(k=1,2,···,n)

b_1+b_2+\cdots+b_{n-1}+2b_n

的最小值

7。設正整數

n\geq1

,實數

a_1,a_2,\cdots,a_n

,證明:

\frac{3[\sum_{k=1}^n(2k-n-1)a_k]^2}{n^2-1}+(\sum_{k=1}^na_k)^2\leq n(\sum_{k=1}^na_k^2)

8。設

a_i\in R^+(1\leq i\leq n)

,證明:

\frac{1}{\sqrt{1+a_1^2+a_2^2}}+\frac{1}{\sqrt{1+a_1^2+a_2^2+a_3^2}}+···+\frac{1}{\sqrt{1+a_1^2+a_2^2+···+a_n^2}}\leq \frac{1}{2\sqrt{2}-1}(1+{\frac{1}{a_1}}+{\frac{1}{a_2}}+···+{\frac{1}{a_n}})

9。設

0<a_n<a_{n-1}<···<a_0=1

,證明:

\sum_{i=1}^n\frac{a_i^2}{a_{i-1}-a_i}>\frac{1}{2}\sum_{i=1}^nia_i-1

10。給定正整數

n\geq2

1\leq x_1\leq x_2\leq\cdots\leq x_n

,證明:

\sum_{k=1}^n\frac{x_k}{1+x_1+x_2+···+x_k}<\sqrt{\sum_{k=1}^n\frac{kx_k}{k+1}}

11。(國子學)給定正整數

n

以及正實數

m

,已知

x_1,x_2,\cdots,x_n

為非負實數且均不大於

m

,求證

(\sum_{i=1}^n\sqrt{1+x_i})^2\leq n^2+n\sum_{i=1}^nx_i-\frac{\sum_{1\leq i<j\leq n}(x_i-x_j)^2}{4(m+1)}

12。(Romania TST 2013) 已知正整數

n

x_1,x_2,\cdots,x_n

為正實數,求

min\left\{ \begin{array}  (x_1,\frac{1}{x_1}+x_2,\frac{1}{x_2}+x_3,···,\frac{1}{x_{n-1}}+x_n,\frac{1}{x_n}\end{array} \right\}

的最大值與

max\left\{ \begin{array}  (x_1,\frac{1}{x_1}+x_2,\frac{1}{x_2}+x_3,···,\frac{1}{x_{n-1}}+x_n,\frac{1}{x_n}\end{array} \right\}

的最小值

13。(2013 Austria MO)非負實數

a_1,\cdots,a_n

滿足對所有的正實數

x_1>x_2>\cdots>x_n

,且

\sum_{i=1}^{n}{x_i}<1

均有

\sum_{k=1}^{n}{a_kx_k^3}<1

,求

na_1+(n-1)a_2+\cdots+a_n

的最大可能值

14。(《Problems from the Book》)定義2n個正實數

a_1,a_2,\cdots,a_n,x_1,x_2,\cdots,x_n

滿足

\sum_{1\leq i< j\leq n}x_ix_j= \binom{n}{2}

,證明:

\sum_{i=1}^n(\frac{a_i}{\sum_{j\ne i}a_j}\sum_{j\ne i}x_j)\geq n

15。(《Problems from the Book》)求最大常數

C

,使得對任意非負實數

x_1,x_2,\cdots,x_n

,都有

\left(\sum_{i=1}^nix_i \right)\left(\sum_{i=1}^{n}{x_{i}^2}\right)\geq C\left(\sum_{i=1}^{n}{x_{i}}\right)^{3}\

16。(1)對於非負實數

a_1,a_2,\cdots,a_n,

證明:

\sum_{k=1}^{n}(\frac{a_1+a_2+...+a_k}{k})^2\leq 4\sum_{k=1}^{n}a_n^2

(2)證明著名的

Hardy不等式:

對於非負實數列

a_1,a_2,\cdots,a_n,\cdots

,則

\sum_{n=1}^{\infty}(\frac{a_1+a_2+...+a_n}{n})^p\leq (\frac{p}{p-1})^p\sum_{n=1}^{\infty}a_n^p

,且證明

(\frac{p}{p-1})^p

是最佳常數(提示:參考例二)

17。(冷崗松)設正整數

n

,實數

x_1,x_2,···,x_n

滿足

x_1^2+x_2^2+···+x_n^2=1

,求證:

\sum_{ij\leq n}x_ix_j\leq2\sqrt{n}

(提示:利用均值不等式配係數)

18。(Fritz Carlson)對於實數

a_1,a_2,\cdots,a_n

,我們有

\pi^2(\sum_{i=1}^na_i^2)(\sum_{i=1}^ni^2a_i^2)\geq (\sum_{i=1}^na_i)^4

19。(西西)給定整數

n\geq 2

,實數

x_1,x_2,\cdots,x_n

,滿足

\left\{ \begin{array}   xx_1+x_2+\cdots+x_n=0 &\\ x_1^2+x_2^2+\cdots+x_n^2=n(n-1)\end{array} \right.

x_1^3+x_2^3+\cdots+x_n^3

的最大值

20。給定正奇數

n\geq3

,求最小的正實數

c

,使得對所有和為1的實數

x_1,x_2,\cdots,x_n

,我們有

c\left(\sum_{i=1}^{n}{x_i^2}\right)^3 \geq \left(\sum_{i=1}^{n}{|x_{i+1}-x_i|}\right)^2\left(\sum_{1\leq i<j\leq n}^{}{(x_i-x_j)^2}\right)

,其中

x_{n+1}=x_1

(提示:最好不要先猜取等)

21。(西西)給定正整數

n

,求最大的實數

c

,使得對於任意的

n

個正實數

a_1,a_2,\cdots,a_n

滿足

\prod_{k=1}^{n}a_k=1

,均有:

 \sum_{k=1}^{n}{\left[\frac{1}{k}-\frac{2}{n(n+1)}\right]}\cdot a_k\geq c\left(\sum_{k=1}^{n}{\frac{k^2}{a_k}}\right)^{\frac{1}{n-1}}\\

(完)如果有需要的話,以後我可能會專門水一篇習題的解答和思路分析的文章(咕咕咕)。本文算是我筆記的一個整理,大部分分析都是我自己口胡的,求大佬們輕噴qwq